Difference between revisions of "2004 AMC 8 Problems/Problem 14"

m (Solution 2)
m (Solution 1)
 
(5 intermediate revisions by 2 users not shown)
Line 18: Line 18:
 
<math>\textbf{(A)}\ 15\qquad \textbf{(B)}\ 18\frac12 \qquad \textbf{(C)}\ 22\frac12 \qquad \textbf{(D)}\ 27 \qquad \textbf{(E)}\ 41</math>
 
<math>\textbf{(A)}\ 15\qquad \textbf{(B)}\ 18\frac12 \qquad \textbf{(C)}\ 22\frac12 \qquad \textbf{(D)}\ 27 \qquad \textbf{(E)}\ 41</math>
  
==Solution==
+
==Solution 1==
Assign points to each of the four vertices and use the shoelace theorem to find the area.
+
 
Letting the bottom left corner be <math>(0,0)</math>, counting the boxes, the points would be <math>(4,0),(0,5),(3,4),</math> and <math>(10,10)</math>. Applying the [[Shoelace Theorem]],
+
<asy>
 +
unitsize(5mm);
 +
defaultpen(linewidth(.8pt));
 +
dotfactor=2;
 +
 
 +
for(int a=0; a<=10; ++a)
 +
for(int b=0; b<=10; ++b)
 +
{
 +
  dot((a,b));
 +
};
 +
 
 +
draw((4,0)--(0,5)--(3,4)--(10,10)--cycle);
 +
draw((0,0)--(10,0)--(10,10)--(3,4)--(0,5)--cycle);
 +
draw((10,4)--(0,4)--cycle);
 +
 
 +
dot("$A$", (0,5), W);
 +
dot("$B$", (3,4), N);
 +
dot("$C$", (10,10), NE);
 +
dot("$D$", (0,4), W);
 +
dot("$E$", (10,4), E);
 +
dot("$F$", (0,0), SW);
 +
dot("$G$", (10,0), SE);
 +
dot("$H$", (4,0), S);
 +
</asy>
 +
 
 +
Divide the shape up as above.
 +
<cmath>Area = [DEGF] + [ABD] + [BCE] - [AFH] - [CGH] = 4 \cdot 10 + \frac12 \cdot 1 \cdot 3 + \frac12 \cdot 7 \cdot 6 - \frac12 \cdot 5 \cdot 4 - \frac12 \cdot 6 \cdot 10 = 40 + \frac32 + 21 - 10 - 30 = \boxed{\textbf{(C)}\ 22\frac12}</cmath>
 +
 
 +
~[https://artofproblemsolving.com/wiki/index.php/User:Isabelchen isabelchen]
 +
 
 +
==Solution 2==
 +
 
 +
Let the bottom left corner be <math>(0,0)</math>. The points would then be <math>(4,0),(0,5),(3,4),</math> and <math>(10,10)</math>. Applying the [[Shoelace Theorem]],
  
 
<cmath>\text{Area} = \frac12 \begin{vmatrix} 4 & 0 \\ 0 & 5 \\ 3 & 4 \\ 10 & 10 \end{vmatrix} = \frac12 |(20+30)-(15+40+40)| = \frac12 |50-95| = \boxed{\textbf{(C)}\ 22\frac12}</cmath>
 
<cmath>\text{Area} = \frac12 \begin{vmatrix} 4 & 0 \\ 0 & 5 \\ 3 & 4 \\ 10 & 10 \end{vmatrix} = \frac12 |(20+30)-(15+40+40)| = \frac12 |50-95| = \boxed{\textbf{(C)}\ 22\frac12}</cmath>
  
==Solution 2==
+
==Solution 3==
Apply [[Pick's Theorem]] on the figure, and you will get 5/2+21-1 which = <cmath>\boxed{\textbf{(C)}\ 22\frac12}</cmath>
+
The figure contains <math>21</math> interior points and <math>5</math> boundary points. Using [[Pick's Theorem]], the area is <cmath>21+\frac{5}{2}-1=\boxed{\textbf{(C)}\ 22\frac12}</cmath>
  
 
==See Also==
 
==See Also==
 
{{AMC8 box|year=2004|num-b=13|num-a=15}}
 
{{AMC8 box|year=2004|num-b=13|num-a=15}}
 
{{MAA Notice}}
 
{{MAA Notice}}

Latest revision as of 10:13, 19 January 2024

Problem

What is the area enclosed by the geoboard quadrilateral below?

[asy] unitsize(3mm); defaultpen(linewidth(.8pt)); dotfactor=2;  for(int a=0; a<=10; ++a) for(int b=0; b<=10; ++b)  {   dot((a,b));  };  draw((4,0)--(0,5)--(3,4)--(10,10)--cycle); [/asy]

$\textbf{(A)}\ 15\qquad \textbf{(B)}\ 18\frac12 \qquad \textbf{(C)}\ 22\frac12 \qquad \textbf{(D)}\ 27 \qquad \textbf{(E)}\ 41$

Solution 1

[asy] unitsize(5mm); defaultpen(linewidth(.8pt)); dotfactor=2;  for(int a=0; a<=10; ++a) for(int b=0; b<=10; ++b)  {   dot((a,b));  };  draw((4,0)--(0,5)--(3,4)--(10,10)--cycle); draw((0,0)--(10,0)--(10,10)--(3,4)--(0,5)--cycle); draw((10,4)--(0,4)--cycle);  dot("$A$", (0,5), W); dot("$B$", (3,4), N); dot("$C$", (10,10), NE); dot("$D$", (0,4), W); dot("$E$", (10,4), E); dot("$F$", (0,0), SW); dot("$G$", (10,0), SE); dot("$H$", (4,0), S); [/asy]

Divide the shape up as above. \[Area = [DEGF] + [ABD] + [BCE] - [AFH] - [CGH] = 4 \cdot 10 + \frac12 \cdot 1 \cdot 3 + \frac12 \cdot 7 \cdot 6 - \frac12 \cdot 5 \cdot 4 - \frac12 \cdot 6 \cdot 10 = 40 + \frac32 + 21 - 10 - 30 = \boxed{\textbf{(C)}\ 22\frac12}\]

~isabelchen

Solution 2

Let the bottom left corner be $(0,0)$. The points would then be $(4,0),(0,5),(3,4),$ and $(10,10)$. Applying the Shoelace Theorem,

\[\text{Area} = \frac12 \begin{vmatrix} 4 & 0 \\ 0 & 5 \\ 3 & 4 \\ 10 & 10 \end{vmatrix} = \frac12 |(20+30)-(15+40+40)| = \frac12 |50-95| = \boxed{\textbf{(C)}\ 22\frac12}\]

Solution 3

The figure contains $21$ interior points and $5$ boundary points. Using Pick's Theorem, the area is \[21+\frac{5}{2}-1=\boxed{\textbf{(C)}\ 22\frac12}\]

See Also

2004 AMC 8 (ProblemsAnswer KeyResources)
Preceded by
Problem 13
Followed by
Problem 15
1 2 3 4 5 6 7 8 9 10 11 12 13 14 15 16 17 18 19 20 21 22 23 24 25
All AJHSME/AMC 8 Problems and Solutions

The problems on this page are copyrighted by the Mathematical Association of America's American Mathematics Competitions. AMC logo.png